Ejercicio 9b

Ejercicio 9b

de Ivana Castillo Rivas -
Número de respuestas: 3

Hola, me está costando calcular la varianza del estimador, llegué a algo pero no creo que este bien. El estimador me dio tita = 2Xn, igual a como lo habíamos hecho en el teórico, así que no creo que tenga que ver con eso. Como puedo calcular la varianza entonces?

En respuesta a Ivana Castillo Rivas

Re: Ejercicio 8b

de Agustín Torres Mari -
A mi me pasa lo mismo con el sesgo, me queda E(2Xn) - tita, luego saco el 2Xn y me queda 2Xn - tita, no se me ocurre como terminarlo.
En respuesta a Agustín Torres Mari

Re: Ejercicio 8b

de Valeria Goicoechea -

Hola, 

Bien. Como bien dices, el estimador de \( \theta \) por el método de los momentos es \( 2 \overline{X}_n \), entonces calcularemos la varianza de \( \hat{\theta}_n = 2 \overline{X}_n \):

Antes que nada, recordar que \( var(aX) = a^2 var(X) \) y que si \( X \) e \( Y \) son variables aleatorias independientes, entonces \( var(X+Y) = var(X)+var(Y) \). Entonces:

\( var(\hat{\theta}_n) = var \left( \frac{2}{n} \sum_i X_i \right) = \frac{4}{n^2} \sum_i var(X_i) \)

Recordar ahora que si \( X \sim U(a,b) \), entonces \( var(X) = \frac{(b-a)^2}{12} \), así que en este caso \( var(X_i) = \frac{\theta^2}{12} \)

\( var(\hat{\theta}_n) = \frac{4}{n^2} \sum_i \frac{\theta^2}{12}= \frac{4}{n^2} n \frac{\theta^2}{12}= \frac{\theta^2}{3n}. \)


Saludos